Solve the following equation
1x + 4 = 3x + 6
X=

Answers

Answer 1
1x + 4 = 3x + 6
X = -1
Answer 2

Answer:

x= -1

Step-by-step explanation:

1x+4=3x+6

x+4=3x+6

x+4-4=3x+6-4

x=3x+6-4

x=3x+2

x-3x=3x+2-3x

-2x=2

-2x=2

-2/-2=2/-2

x= -1


Related Questions

0.612 repeating as a fraction

Answers

Answer:

153/250

Step-by-step explanation:

did this in 4th grade

Brown Law Firm collected data on the transportation choices of its employees for their morning commute. The table shows the percentages of the type of transportation of the male and female employees. A 5-column table has 3 rows. The first column has entries Male, female, total. The second column is labeled public with entries 12, 8, 20. The third column is labeled Own with entries 20, 10, 30. The fourth column is labeled other with entries 4, 6, 10. The fifth column is labeled total with entries 36, 24, 60. Consider the following events: A: The employee is male. B: The employee is female. C: The employee takes public transportation. D: The employee takes his/her own transportation. E: The employee takes some other method of transportation. Which two events are independent? A and C A and D B and D B and E.

Answers

The two events out of the listed events which are independent events are given by: Option A: A and C

What is chain rule in probability?

For two events A and B, by chain rule, we have:

[tex]P(A \cap B) = P(B)P(A|B) = P(A)P(A|B)[/tex]

What is law of total probability?

Suppose that the sample space is divided in n mutual exclusive and exhaustive events tagged as

[tex]B_i \: ; i \in \{1,2,3.., n\}[/tex]

Then, suppose there is event A in sample space.

Then probability of A's occurrence can be given as

[tex]P(A) = \sum_{i=1}^n P(A \cap B_i)[/tex]

Using the chain rule, we get

[tex]P(A) = \sum_{i=1}^n P(A \cap B_i) = \sum_{i=1}^n P(A)P(B_i|A) = \sum_{i=1}^nP(B_i)P(A|B_i)[/tex]

How to form two-way table?

Suppose two dimensions are there, viz X and Y. Some values of X are there as [tex]X_1, X_2, ... , X_n[/tex] values of Y are there as [tex]Y_1, Y_2, ..., Y_k[/tex]rows and left to the columns. There will be [tex]n \times k[/tex]values will be formed(excluding titles and totals), such that:

Value([tex]i^{th}[/tex] row, [tex]j^{th}[/tex] column) = Frequency for intersection of [tex]X_i[/tex] and [tex]Y_j[/tex]values are going in rows, and Y values are listed in columns).

Then totals for rows, columns, and whole table are written on bottom and right margin of the final table.

For n = 2, and k = 2, the table would look like:

[tex]\begin{array}{cccc}&Y_1&Y_2&\rm Total\\X_1&n(X_1 \cap Y_1)&n(X_1\cap Y_2)&n(X_1)\\X_2&n(X_2 \cap Y_1)&n(X_2 \cap Y_2)&n(X_2)\\\rm Total & n(Y_1) & n(Y_2) & S \end{array}[/tex]

where S denotes total of totals, also called total frequency.

n is showing the frequency of the bracketed quantity, and intersection sign in between is showing occurrence of both the categories together.

How to calculate the probability of an event?

Suppose that there are finite elementary events in the sample space of the considered experiment, and all are equally likely.

Then, suppose we want to find the probability of an event E.

Then, its probability is given as:

[tex]P(E) = \dfrac{\text{Number of favorable cases}}{\text{Number of total cases}} = \dfrac{n(E)}{n(S)}[/tex]

where favorable cases are those elementary events who belong to E, and total cases are the size of the sample space.

How to find if two events are independent?

Suppose that two events are denoted by A and B.

They are said to be independent event if and only if:

[tex]P(A \cap B) = P(A)P(B)[/tex]

The given frequency table is:

[tex]\begin{array}{ccccc} &\text{Public}&\text{Own}&\text{Others}&\text{Total}\\\text{Male}&12&20&4&36\\\text{Female}&8&10&6&24\\\text{Total}&20&30&10&60\end{array}[/tex]

The probability table for the same labels would be:

[tex]\begin{array}{ccccc} &\text{Public}&\text{Own}&\text{Others}&\text{Total}\\\text{Male}&12/60&20/60&4/60&36/60\\\text{Female}&8/60&10/60&6/60&24/60\\\text{Total}&20/60&30/60&10/60&1\end{array}[/tex]

The events A, B,C,D and E are given as:

A: The employee is male. B: The employee is female. C: The employee takes public transportation. D: The employee takes his/her own transportation.E: The employee takes some other method of transportation.

Checking all the listed options one by one, for them being independent:

Case 1: A and C

P(A) = P(The employee is male. ) = 36/60

P(C) = P(The employee takes public transportation.) = 20/60[tex]P(A \cap C) = 12/60 \\\\ P(A)P(C) = \dfrac{36 \times 20}{60^2} = 12/60[/tex]

[tex]P(A \cap C) = P(A)P(C)[/tex]

Thus, A and C are independent events.

Case 2: A and D

P(A) = P(The employee is male. ) = 36/60

P(D) = P(The employee takes his/her own transportation.) = 30/60

[tex]P(A\cap D) = 20/60\\\\P(A)P(D) = \dfrac{30 \times 36}{60^2} = 12/60 \neq P(A \cap D)[/tex]

Thus, A and D are not independent events.

Case 3: B and D

P(B) = P(The employee is female. ) = 24/60

P(D) = P(The employee takes his/her own transportation.) = 30/60

[tex]P(B \cap D) = 10/60 \neq P(B)P(D)=\dfrac{24 \times 30}{60^2} = 12/60[/tex]

Thus, B and D are not independent events.

Case 4: B and E

P(B) = P(The employee is female. ) = 24/60

P(E) = P(The employee takes some other method of transportation.) = 10/60

[tex]P(B \cap E) = 6/60 \neq P(B)P(E)= \dfrac{24 \times 10}{60^2} = 4/60[/tex]

Thus, B and E are not independent events.

Thus, the two events out of the listed events which are independent events are given by: Option A: A and C

Learn more about independent events here:

https://brainly.com/question/3898488

Answer:

First option!!

"A and C"

Step-by-step explanation:

Edge 2022

The table shows values for functions f(x) and g(x). X f(x)=0. 25x g(x)=−0. 75x 1 −3 64 3. 25 −2 16 2. 5 −1 4 1. 75 0 1 1 1 0. 25 0. 25 2 0. 0625 −0. 5 3 2000. 015625 −1. 25 What is the solution to f(x)=g(x) ? Select each correct answer. X=−3 x=−2 x=−1 x = 0 x = 1 x = 2 x = 3.

Answers

Solution to f(x)=g(x) will be x=0 and x=1 because functional value on these two points is same for both the functions.

it is given that

[tex]f(x)= 0.25^x\\g(x)= -0.75x+1[/tex]

given table is,

x    f(x)     g(x)

-3    64     3.25

-2    16      2.5

-1      4       1.75

0      1         1

1     0.25    0.25

2    0.625   -0.5

3    0.015625   -1.25

what is the solution of two functions?

solutions of two functions are the points where two functions intersect or we can say that points where the values of two functions is equal.

as we can see that for x=0 and for x=1, the value of functions i.e. f(0)=g(0)=1 and f(1)=g(1)=0.25

it means these two functions will intersect at x=0 and x=1 so these values(0 and 1) of x will be the solution to f(x)=g(x)

therefore, solution to f(x)=g(x) will be x=0 and x=1

to get more about solutions of two functions refer to:

https://brainly.com/question/1306680

if it jake 2/3 hours to paint 2/5 of his room, how long would it take to paint his sister's room?

Answers

assuming his room and his sisters room are the same size
(2/3)/2=1/3 hour to paint 1/5 of his room
(1/3) x 5= 5/3 hours to pain the entire room

hehehehehehhelphelphelpgelpggepkepewnwe

Answers

(6,5)

If you start at the origin (which is (0,0)) and go 6 right, that's six for x. If you then go 5 up, that means y is 5.

(6,5)

Answer:

(6,5)

Step-by-step explanation:

6 is on the x-axis, and 5 is on the y-axis

I need help with these two questions please !

Answers

Answer:

9. 60

10. 12

Step-by-step explanation:

9.

We need to find the least common multiple (LCM) of 12 and 15.

Here are two methods:

Method 1

List several multiples of each number until you find the smallest number that is a multiple of both.

Multiples of 12: 12, 24, 36, 48, 60, 72

Multiples of 15: 15, 30, 45, 60, 75

60 is the LCM of 12 and 15.

Answer: 60

Method 2

Find the prime factorization of both numbers.

12 = 2² × 3

15 = 3 × 5

The LCM is the product of the following factors:

Common and not common prime factors with greater exponent.

2 is not common. Use 2².

3 is common. Use 3.

5 is not common. Use 5.

LCM = 2² × 3 × 5 = 60

Answer: 60

10.

We need the LCM of 4 and 6.

I'll use the second method above which is prime factorization.

4 = 2²

6 = 2 × 3

LCM = 2² × 3 = 12

Answer: 12

evaluate the function when x=4

Answers

The graph of the function is defined in the attached file please find it.

Graph function:

[tex]\to \bold{x=4}[/tex]

In the given question, the x-axis holds a value that is "4".The holding value is "4" which is a positive number, and in the graph, it represents the positive value, which is defined in the attached file please find the attached file.

Find out more information about the function here:

brainly.com/question/12406000

Write a quadratic function whose graph has a vertex of (3,2) and passes through the point (4,7).
f(x) =_

Answers

Answer:x^4 +y+1

Step-by-step explanation:

En una ciudad de 270000 habitantes las 5/9 partes son mujeres ¿cuantas mujeres y cuantos hombres hay?

Answers

270,000 * [tex]\frac{5}{9}[/tex] = 150,000 women / mujeres

270,000 - 150000 = 120000 men / hombres

Have a nice day!

     I hope this is what you are looking for, but if not - comment! I will edit and update my answer accordingly.

- Heather

/

¡Que tenga un lindo día!

Espero que esto sea lo que estás buscando, pero si no, ¡comenta! Editaré y actualizaré mi respuesta en consecuencia.

- Heather

I WILL GIVE BRAINLIEST
Dan’s dog-walking job pays $15 per hour. His job as a car-wash attendant pays $400 each week. Dan wants to know how many hours he needs to spend walking dogs to earn more than $520 in a week.

Which three inequalities can model this situation?

Select all the correct answers.

A. x(15 + 400) > 520
B. 520 < 400 + 15x
C. 15x > 120
D. 15x + 520 > 400
E. 520 < 15x + 400x
F. 15x + 400 > 520

Answers

Answers ⤵️

520<400+15x15x>12015x+400>520

[tex]\large\underline{\underline{\maltese{\purple{\pmb{\sf{\: Explanation :-}}}}}}[/tex]

Let the number of hours be x for his job of Dog walking..

So, The equation would be:

[tex] \blue \dashrightarrow \green{ \sf15x +400}[/tex]

Dan wants to earn more than $520 in a week, so this means that the expression is greater than 520:

[tex] \blue \dashrightarrow \green{ \sf15x + 400 > 520}[/tex]

If we move the constant to the right-hand side and change its sign, The equation would be:

[tex] \blue \dashrightarrow \green{ \sf15x > 520 - 400} \\ \blue \dashrightarrow \green{ \sf15x > 120}[/tex]

Also, the first inequality can be rearranged as:

[tex] \blue \dashrightarrow \green{ \sf520 < 400 + 15x}[/tex]

help me. I need help

Answers

Answer:

3 ------------- √9

6 ------------- ³√216

7 ------------- ³√343

8 ------------- √64

9 ------------ ³√729

10 ----------- √100

Step-by-step explanation:

Hope this helps .

Answer:

[tex]1. \sqrt{9} =3[/tex]

[tex]2. \sqrt[3]{729}=9[/tex]

[tex]3. \sqrt{100} =10[/tex]

[tex]4. \sqrt[3]{216} =6[/tex]

[tex]5. \sqrt{64} =8[/tex]

[tex]6. \sqrt[3]{343} = 7[/tex]

Step-by-step explanation:

[tex]1. \sqrt{9} =3[/tex]    [tex]3*3=9[/tex]

[tex]2. \sqrt[3]{729}=9[/tex]

[tex]\sqrt[3]{9^3}=9[/tex]

[tex]3. \sqrt{100} =10[/tex]    [tex]10*10=100[/tex]

[tex]4. \sqrt[3]{216} =6[/tex]

[tex]\sqrt[3]{6^3}=6[/tex]

[tex]5. \sqrt{64} =8[/tex]     [tex]8*8=64[/tex]

[tex]6. \sqrt[3]{343} = 7[/tex]

[tex]\sqrt[3]{7^3}=7[/tex]

Hope this helps!

10. Which is equivalent to 4/3?
a.
1218
.
b. 3/32
31
.
c. 3/20
d. 35/

Answers

Answer:

3/30

Step-by-step explanation:

What do the grid boxes in a punnett square represent?.

Answers

Answer:

All the possible allele combinations in the offspring

Step-by-step explanation:

Hope this helps!

Help!!!! Its A Math Question Which of the following equations describes the relationship for the values in the table.

Question options:

y = 10x−3

y=2x2−1

y=2x2+1

y=x2+3

Answers

Answer:

[tex]y = 10 \times - 1 = [/tex]

[tex] \times - 1 = - 10[/tex]

y=

[tex] - 10[/tex]

I NEED HELP BAD PLEASE HELP ME!!!!!!
Many bank accounts never go below zero. But some banks will allow a negative balance, at least for a short time, called an overdraft. It means someone has taken out, or 'drafted', more money than was in the account to begin with. Mila's account went into overdraft. To get back to a positive balance, she deposited money at a steady rate of $39.92 per week. After 3 weeks, she had $93.29 in the account. What was the balance when the account went into overdraft?

Answers

Answer:

The balance when the account went into overdraft was -38.02

Explanation:

Let x be the balance when the account went into overdraftTo get back to a positive balance he deposited money at a steady rate of $20.06 per week. Amount deposited per week = $20.06Amount deposited 8 weeks = Now amount in account after 8 weeks =x+160.48We are given that After 8 weeks, he had $122.46 in the account.So,x+160.48=122.46x=122.46-160.48x=-38.02

One leg of a 45°-45°-90° triangle measures 6 inches. What is the length of the hypotenuse?

Answers

This is a right triangle (hence the 90° angle), so you'd find the legs (both are the same length of a right triangle) by using Pythagoras Theorem (a2 + b2 = c2) Since you have the hypotenuse (6"), just go backwards. 62 = 36 = a2 + b2. Like I said before the legs are the same length so you can rewrite it as 36 = 2x2, where x = either a or b. Then just solve for x.

Answer:

[tex]6\sqrt{2}[/tex]

Step-by-step explanation:

In a 45-45-90 triangle, the legs are x, and the hypotenuse is [tex]x\sqrt{2}[/tex]

Since x, in this case, is 6, the hypotenuse is [tex]6\sqrt{2}[/tex]

Hope this helps!

If the discriminant of an equation is positive, which of the following is true of
the equation?
A. It has one real solution.
B. It has two complex solutions.
C. It has one complex solution.
ОО
D. It has two real solutions.
SUBMIT

Answers

Answer:

D

Step-by-step explanation:

Two real and distinct roots

Answer:

D

Step-by-step explanation:

The conditions for the discriminant and the roots of the equation are

• If b² - 4ac > 0 then 2 real and distinct roots

• If b² - 4ac = 0 then 2 real and equal roots

• If b² - 4ac < 0 then roots are complex

here the discriminant is positive , that is greater than zero , then

the equation has two real solutions D

Help please and thank you giving brainliest. Please don’t spam answer just for point :)

Answers

Answer:

x ( [tex]\frac{1}{2}[/tex] b, [tex]\frac{1}{2}[/tex] c )

Step-by-step explanation:

x is the midpoint of PQ and its coordinates are the average of the coordinates of the endpoints P and Q , then

x = ([tex]\frac{b-0}{2}[/tex], [tex]\frac{c-0}{2}[/tex]  ) = ( [tex]\frac{1}{2}[/tex] b, [tex]\frac{1}{2}[/tex] c )

Please tell me the mean median and mode of
82, 78, 82, 80, 100, 96, 75, 91

Answers

Answer:

90 is the median and 86 is the mean

Step-by-step explanation:

:)

The median of any set of numbers is the middle number of the set when then all the numbers are arranged from smallest to greatest. However, if there is an even number of numbers in the set, just take the average of the two numbers close to the middle

75, 78, 80, 82, 82, 91, 96, 100[tex]median = \frac{82+82}{2} =\frac{164}{2} =82[/tex]

The mode of any set is the number that shows up the most often in the set

mode = 82 (because it shows up twice while other numbers only show up once)

The mean of any set is all the numbers in the set divided by the number of numbers in the set

[tex]mean = \frac{82 + 78+82+80+100+96+75+91}{8} =\frac{684}{8} =85.5[/tex]

The range is the smallest number of the set subtracted from the largest number of the set.

[tex]range = 100 - 75 = 25[/tex]

Hope that helped!

 

last saturday angela had a birthday party. 9 people had to leave early. there were 14 people left. how many people showed up for her party?

Answers

Answer:

23 people showed up for Angela's party.

Step-by-step explanation:

Hi there!

We're given that 9 people at the party had to leave early, and there are 14 people left.

Let n be equal to the original number of people who showed up for Angela's party.

n - 9 = 14

n = 14 + 9

n = 23

Therefore, 23 people showed up at her party.

I hope this helps!

23 people showed up for her party

How to determine the number of people in the party

The given parameters are:

People that left early = 9

People remaining = 14

Represent the number of people in the party with x.

So, the value of x is calculated as:

x =14 + 9

Evaluate the sum

x = 23

hence, 23 people showed up for her party

Read more about the sum operator at:

https://brainly.com/question/4344214

Tim thought of a number, x. he multiplied x by 15:and then subtracted 3. and then Tim told Jim the result, y.

Write a formula that will help Jim quickly find Tim’s number if he knows the value of y.

Answers

Answer: x = (y+3)/15.

Step-by-step explanation:

We know that 15x - 3 = y.

We can add 3 to both sides:

15x = y + 3

We can divide both sides by 15:

x = (y+3)/15.

Therefore, if given y, we can find x by computing (y+3)/15.

Ya'll FR get 50 points if you answer this.
Why is the graph shown a line of best fit?
A) The graph shows a line of best fit because the association is zero.
B) The graph shows a line of best fit because the points fit around the line.
C) The graph shows a line of best fit because there is a straight line.
D) The graph shows a line of best fit because the points are plotted evenly above and below the line.

Answers

Answer:

I would say B

Step-by-step explanation:
A. would make no sense and the graph will always have a straight line unless its compairing something and it does not have anything to do with 0 because it shows where it starts

The person is correct A would be the most accurate answer

4/1 = 12/3 so 12 cups =_____ quarts

Answers

Answer:

3 quarts

Step-by-step explanation:

4/1 = 12/3 so 12 cups =3 quarts

Circle O has a circumference of 88 cm.
What is the length of the radius of the circle?
cmין

Answers

Answer:

r = 14 cm

Step-by-step explanation:

Circumference = 2πr = 88

=> r = 88 / 2 x 22/7

=> r = 88 x 7 / 44

=> r = 2 x 7

=> r = 14 cm

Which of the following is the total number of pennies on Rows 1-4 (the first 32 squares)? 232 – 1 232 232 1.

Answers

The total number of pennies on Rows 1-4 (the first 32 squares) is 4,294,967,295 or 2³².

What is geometric progression?

A geometric progression is the series of numbers such that the ratio of any two consecutive numbers of the series is the same.

If we look closely at the problem we will understand that on each of the square boxes the number of pennies will get double from the previous one therefore, we can say that it is forming a geometric progression where the first term of the sequence is 1, while the common ratio is of 2. thus,

The total number of pennies on Rows 1-4 (the first 32 squares) is the sum of the geometric progression for the first 32, terms, therefore,

[tex]S_n = a_1(\dfrac{r^n-1}{r-1})[/tex]

Now, as we know that the first term of the series is 1, while the common ratio(r) is 2, and the number of terms(n=32). Thus, the sum can be written as,

[tex]S_n = a_1(\dfrac{r^n-1}{r-1})\\\\S_n = 1(\dfrac{2^{32}-1}{2-1})\\\\S_n = 4,294,967,295[/tex]

The number can also be written as,

[tex]S_n = 4,294,967,295\\\\S_n = 2^{32}[/tex]

Hence, the total number of pennies on Rows 1-4 (the first 32 squares) is 4,294,967,295 or 2³².

Learn more about Geometric progression:

https://brainly.com/question/14320920

Answer:

A: 2^32 – 1

Step-by-step explanation:

David saves $25.78 to buy a video game. After he buys the video game, he has $3.04 left over. How much does David spend on the video game?

Answers

Answer:

He spent 22.74

Step-by-step explanation:

because 25.78−22.74 = 3.04

B. Translate each word into algebraic eqaution. (Use any letter)

1. Twice a number is 18. _____
2. The difference of a number and 5 is equal to 8. _____
3. The quotient of a number and 7 is 21. _____
4. Thrice a number plus 5 is 12. _____
5. The product of two and a number is 18. ______

Help me in Mathematics ASAP ASAP​ ASAPPP

Answers

Answer

1. 9

1. 13

1. 147

1. 7/3

1. 9

Step-by-step explanation:

Algebraic equations

1. 2x=18

2. x-5=8

3. (x/7)=21

4. 3x+5=12

5. 2x=18

The ages of two girls are in the ratio of 2:3 . 6 years hence the ratio of the girls will be 11 : 13 find their present age .​

Answers

Is the question right?

Juan wants to change the shape of his vegetable garden from a square to a rectangle, but keep the same area so he can grow the same amount of vegetables. The rectangular garden will have a length that is 2 times the length of the square garden, and the width of the new garden will be 16 feet shorter than the old garden. The square garden is x feet by x feet. What is the quadratic equation that would model this scenario? x2 = (2x)(x – 16) x2 = (x)(x – 16) x2 = (x)(x 16) x2 = (2x)(x 16).

Answers

The quadratic equation that would model this scenario is

[tex]x^{2} = 2x(x-16)[/tex]

Let us take the side of the square = x

Area of the square = x²

Length of the rectangular garden = 2x

Width of the rectangular garden = x-16

So, the area of the new vegetable garden = length*width

Area of the new or rectangular vegetable garden = 2x(x-16)

What is a quadratic equation?

The polynomial equation whose highest degree is two is called a quadratic equation. The equation is given by [tex]ax^2+bx+c[/tex]coefficient [tex]x^{2}[/tex]non-zero.

Since it is given that

Area of square garden = area of the rectangular garden

[tex]x^{2} = 2x(x-16)[/tex]

Thus, the quadratic equation that would model this scenario is

[tex]x^{2} = 2x(x-16)[/tex]

To get more about quadratic equations refer to:

https://brainly.com/question/1214333

Answer:

its a

Step-by-step explanation:

help me pleaseeeeeeeee

Answers

Answer:

Angles on a straight line add up to 180 degrees so for the 129 degrees you will take 180 - 129 = 51. In the triangle you will add 51 and 55 = 106 but remember in a triangle all angles should be equal to 180. So 180 - 106 = 74. X=74degrees

In a triangle, the interior angles have a sum of 180 degrees. To determine the value of x, you need to first find the measure of the angle supplementary to the exterior angle.

Since the exterior angle is 129 degrees, the other angle supplementary to it would equal 51.

After you know that, add 51 + 55 = 106

180 - 106 = 74, so the measure of x is 74 degrees.
Other Questions
Which sequence of geologic events occurred after the formation of the cambrian limestone. PLEASE HELP !!!12) Using the Integers from -9 to 9 only one time each, create a system of 3 equations such that the solution is (1.1). 7. What causes greenhouse gasses to absorb heat when gasses like oxygen do not? What jobs are service jobs?Group of answer choicesLawyerFirefighterAll of the abovePolice officerfast answer pls and get 12 brainly points you have been asked to give advice to your friends on how to overcome stereotypes write down three ideas that you will share with them HELP ASAP! 2. in the figure, x is an exterior angle to the triangle below. (a) explain why x is equal to the sum of the measures of the two nonadjacent interior angles. You need to present the equation with the steps/work that proves this theorem. Step 1: __Step 2: __ Step 3: __ (b) what is the measurement of < x ? Answer this question please Look at this square: 8 m 8 m If the side lengths are halved, then which of the following statements about its area will be true? The new area will be 1 of the old area. 4 The new area will be 1 of the old area. 2 The new area will be 1 of the old area. end The new area will be 16 times the old area. If you were running a race and you passed the person in 2nd place, what place would you be in now? Are there any side effects of insulin injection in a long way?. y=182.5^3How do i solve i need help with this What literary device is seen in the lines below?It is not possible: you have not a man in allAthens able to discharge Pyramus but he.A) allusion-mythologyB) anachronismC) allusion-literatureD) allusion-folklore/superstition the reciprocal of each number?2 1/30.6-52 1/21.43.2-4 Which american city has the most populous zip code. Brian longs to play with the other children in his class, but they don't seem to want to play with him. He spends most recesses playing alone in the sandbox, wishing a classmate would join him. Based on this information, Abraham Maslow would claim that Brian is likely preoccupied with his need for __________. A. Self-actualization B. Belongingness C. Safety D. Esteem. 32. Explain why investing $1000 at 4% interest, compounded continuously, for 2 years isequivalent to investing $1000 at 8% interest for 1 year. 19cm 20cm 25cm what is the area of this trapezoid What are the three parts of the missouri compromise. The graph shows the relationship between distance driven and gallons of gas used.